Hanging cable

A telephone cable hangs between two poles of equal height located d = 100 m d=100~\mbox{m} apart. Because of its weight, the cable sags h = 0.7 m h=0.7~\mbox{m} in the middle, that is to say, h h is the vertical distance between the center of the cable and the points of suspension.

If the mass per unit length of the cable is λ = 0.2 kg/m \lambda=0.2~\mbox{kg/m} , find the tension of the cable in Newtons at the lowest point (in the middle).

Details and assumptions

Assume g = 9.8 m / s 2 . g=9.8~\mbox{m}/\mbox{s}^2.


The answer is 3500.

This section requires Javascript.
You are seeing this because something didn't load right. We suggest you, (a) try refreshing the page, (b) enabling javascript if it is disabled on your browser and, finally, (c) loading the non-javascript version of this page . We're sorry about the hassle.

4 solutions

Josh Silverman Staff
Oct 29, 2013

This problem can be defeated by looking at the torque about the lowest point on the cable.

As we move along the cable, the mass that must be supported by the vertical component of the tension is constantly changing. The horizontal component however is constant. Pick any segment of the cable and calculate the net force on it in the horizontal dimension, with any two endpoints p 1 p_1 and p 2 p_2 we must have T x ( p 1 ) = T x ( p 2 ) T_x(p_1) = T_x(p_2) (or else the cable would accelerate in either direction, but alas, it is stationary).

Therefore, it must be that the tension at the lowest point in the wire (which is purely horizontal) must be equal to the horizontal component of the tension at any other point.

We have three torques to consider, the vertical component of the tension at the support, which is equal to half the weight of the wire and acts through the perpendicular distance l / 2 l/2 from the origin (point L in the diagram), the horizontal component of the tension at the support and acts through the perpendicular distance h h to the origin, and the weight of the wire which acts at the average perpendicular distance l / 4 l/4 origin. This is diagrammed below:

Fig 1. Fig 1.

Balancing the torques, we have

λ l 2 8 g + T x h = λ l 2 4 g T x h = λ l 2 8 g T x = λ l 2 8 h g \begin{aligned} \lambda\frac{l^2}{8}g + T_xh &= \lambda\frac{l^2}{4}g \\ T_xh &= \lambda\frac{l^2}{8}g \\ T_x &= \lambda\frac{l^2}{8h}g \end{aligned}

Plugging in the values from the problem, we find that the tension at the low point is 3500 N 3500\mbox{ N} .

It is important to point out that this all assumes the deformation of the wire to be negligible, i.e. ( l 2 ) 2 + h 2 l 2 \sqrt{\left(\frac{l}{2}\right)^2+h^2}\approx \frac{l}{2} , which is true.

Josh Silverman Staff - 7 years, 7 months ago

Log in to reply

amazing solution!

jatin yadav - 7 years, 7 months ago

Log in to reply

I don't get why this comment has ten down votes....

Yuchen Liu - 7 years, 7 months ago

Log in to reply

@Yuchen Liu We have identified that a particular individual has been abusing the system, and are correcting for that.

Calvin Lin Staff - 7 years, 7 months ago

Hi, josh, we had to find tension at lowest point, and hence we would balance torque on half string about the extreme end, and the equation T x p 1 = T x p 2 T_{x_{p_{1}}} = T{x_{p_{2}}} would not have been used.

jatin yadav - 7 years, 7 months ago

Log in to reply

Jatin, I don't think I understand the point you are making. Can you say more about it?

Josh Silverman Staff - 7 years, 7 months ago

Hey Jatin, I think I get what you're saying. Your idea is good, it makes the calculation simpler than by balancing about the low point like I did and it also eliminates the need to make the argument about horizontal tension staying constant along the cable.

Balancing about the extreme side:

T h = l 4 λ g l 2 T = λ g l 2 8 h \displaystyle Th=\frac{l}{4}\lambda g \frac{l}{2}\rightarrow T = \lambda \frac{gl^2}{8h}

Josh Silverman Staff - 7 years, 7 months ago

Log in to reply

Yes, i meant the same. :)

jatin yadav - 7 years, 7 months ago

Nice solution Josh! I did it the same way but instead I balanced forces instead of moments.

I wanted to write a solution but...meh :P

Anyways, here it goes:

Let there be a force R at the ends of cable at an angle θ \theta with the horizontal. Balancing the forces in vertical direction, 2 R sin θ = λ g / 2 2R\sin\theta=\lambda \ell g/2 .

Consider the half portion of cable. As already explained by Josh, the tension at lowest point is completely horizontal. Hence T = R cos θ T=R\cos\theta . Dividing both the equations,

2 tan θ = λ g 2 T \displaystyle 2\tan\theta=\frac{\lambda \ell g}{2T}

Substituting tan θ = 2 h / \tan \theta=2h/\ell and solving for T,

T = λ 2 g 8 h \displaystyle T=\frac{\lambda \ell^2g}{8h}

I hope this a correct approach.

Pranav Arora - 7 years, 7 months ago

Log in to reply

This is correct but incomplete, you should tell how you get t a n θ = 2 h l tan\theta = \frac{2h}{l} , you must be knowing the equation of catenary to get that.

jatin yadav - 7 years, 7 months ago

Log in to reply

Hi Jatin! :)

I thought that since h<<<l/2, it is safe to say that tan θ = 2 h / \tan\theta=2h/\ell .

Is this a wrong assumption?

Pranav Arora - 7 years, 7 months ago

Log in to reply

@Pranav Arora If you take t a n θ = 2 h l tan\theta = \frac{2h}{l} , you get answer as 7000 N 7000N .Actually , t a n θ = 4 h l tan\theta = \frac{4h}{l} , also, the reason why you are getting right answer \is that your equation 2 R s i n θ = λ l g 2 2Rsin\theta = \frac{\lambda lg}{2} is wrong, it should be 2 R s i n θ = λ l g 2Rsin\theta = \lambda lg , as you are balancing force on whole of the string.

Actually, the θ \theta we are using in the equations the one which the tangent to string at extreme end, and the t a n θ tan\theta you are calculating is for the angle which the straight line joining the extreme end and mid makes with the horizontal.

So, i should change my above reply as:

"it is incorrect as well as incomplete".

Hope this helps!

jatin yadav - 7 years, 7 months ago

Log in to reply

@Jatin Yadav Ah yes, you are right Jatin, I guess I was lucky while solving the problem. I really didn't notice that.

Very sorry. x(

Pranav Arora - 7 years, 7 months ago
Cody Johnson
Oct 27, 2013

It is well-known that a hanging rope assumes the shape of a catenary, with equation

y = a cosh x a y=a\cosh\frac{x}{a}

where a = T λ g a=\frac{T}{\lambda g} . For x = 0 x=0 , we have the height a a . For x = d 2 x=\frac{d}{2} , we have the height a cosh d 2 a a\cosh\frac{d}{2a} . But, the problem also tells us that a cosh d 2 a a = h a\cosh\frac{d}{2a}-a=h . So, one can solve for a a in the equation

a ( cosh 50 a 1 ) = 0.7 a(\cosh\frac{50}{a}-1)=0.7

and plug in T = a λ g = ( 1785.83094 m ) ( 0.2 k g / m ) ( 9.81 m / s 2 ) 3500 N T=a\lambda g=(1785.83094m)(0.2kg/m)(9.81m/s^2)\approx\boxed{3500N}

How did you get a as 1785.83094 ??

Santanu Banerjee - 7 years, 7 months ago

Eh.. could you please explain how the equation is derived..? It isn't well-known to me and I could only get the formula after a bit of research. Thanks in advance.

Paul Dane - 7 years, 7 months ago

Log in to reply

You can see a step by step derivation here

David Mattingly Staff - 7 years, 7 months ago

I doubt there's a closed form to the equation involving hyperbolic cosine, it looks transcendental.

A L - 7 years, 7 months ago

How did you get that acosh(d/2a) - a = h? And how did you solve the equation after that?

Dinh Ngoc Duc - 7 years, 7 months ago

The free-hanging cable has a catenary curve of the form:

y = a cosh ( x a ) y = a \cosh { \left( \dfrac {x} {a} \right) }

where the y-axis pass through the lowest point, and

  • a = T 0 λ g a = \dfrac {T_0} {\lambda g}
  • T 0 = T_0 = the tension of the cable at the lowest point
  • λ = 0.2 \lambda = 0.2 kg/m, the mass per unit length of the cable
  • g = 9.8 g = 9.8 m/s 2 ^2

Therefore, y ( 50 ) y ( 0 ) = a cosh ( 50 a ) a = h = 0.7 y(50) - y(0) = a \cosh {\left( \dfrac {50} {a} \right) } - a = h = 0.7

By numerical method, it is found that T 0 = 3500 N T_0 = \boxed {3500} N

Santanu Banerjee
Nov 3, 2013

This is a great problem and I would thank Brilliant for this. I am sure that most of the others must have done this assuming the curve to be a parabola and the radius twice the focus but that is an approximation...

Since the curve will be that of a Catenary passing through (50 , a+0.7) we put these in the equation of catenary and get a .Since we need to find the tension(T) at the lowest point which is equal to the constant horizontal force H=a w (where W is the weight per unit length i.e. 0.2*9.8).

Thus answer is 1785.83 0.2 9.8 = 3500.2268

when we put (50,a+.7) in the equation y=acosh(x/a), and then if we convert it into exponential form. it becomes very complicated to solve. can you please elaborate how did you get a?

Kaustav Mukherjee - 7 years, 7 months ago

0 pending reports

×

Problem Loading...

Note Loading...

Set Loading...